- PowerScore Staff
- Posts: 5972
- Joined: Mar 25, 2011
- Sat Jun 23, 2018 8:55 am
#88201
Complete Question Explanation
(The complete setup for this game can be found here: lsat/viewtopic.php?f=171&p=88197#p88197)
The correct answer choice is (B)
The condition in this question establishes that S is not immediately above O. Because S cannot be immediately above R (from the first rule), and also cannot be immediately above L or M (from the third rule), S must be immediately above V, the only remaining variable.
Because R cannot be consecutive with S, and V, O, L, and M are all below S, no other variable other than S is available to be sixth. Thus, S must be sixth, and from the prior inference V must then be fifth.
With S and V placed as sixth and fifth, the primary uncertainty is the placement of R, O, L, and M. O, L, and M must still adhere to the sequence established in the second and third rules, creating the following scenario:
Because O must always be above the LM block, the LM block must always be 1-2 or 2-3, and thus using Hurdle the Uncertainty we can determine that L or M is always second, and O is always third or fourth:
R can be first, third, or fourth.
Answer choice (A) is incorrect because S must be immediately above V.
Answer choice (B) is the correct answer. R can be fourth, O can be third, M can be second, and L can be first.
Answer choice (C) is incorrect because from the second rule L is immediately below M.
Answer choice (D) is incorrect because L or M must be second.
Answer choice (E) is incorrect because S is the top layer.
(The complete setup for this game can be found here: lsat/viewtopic.php?f=171&p=88197#p88197)
The correct answer choice is (B)
The condition in this question establishes that S is not immediately above O. Because S cannot be immediately above R (from the first rule), and also cannot be immediately above L or M (from the third rule), S must be immediately above V, the only remaining variable.
Because R cannot be consecutive with S, and V, O, L, and M are all below S, no other variable other than S is available to be sixth. Thus, S must be sixth, and from the prior inference V must then be fifth.
With S and V placed as sixth and fifth, the primary uncertainty is the placement of R, O, L, and M. O, L, and M must still adhere to the sequence established in the second and third rules, creating the following scenario:
Because O must always be above the LM block, the LM block must always be 1-2 or 2-3, and thus using Hurdle the Uncertainty we can determine that L or M is always second, and O is always third or fourth:
R can be first, third, or fourth.
Answer choice (A) is incorrect because S must be immediately above V.
Answer choice (B) is the correct answer. R can be fourth, O can be third, M can be second, and L can be first.
Answer choice (C) is incorrect because from the second rule L is immediately below M.
Answer choice (D) is incorrect because L or M must be second.
Answer choice (E) is incorrect because S is the top layer.
You do not have the required permissions to view the files attached to this post.
Dave Killoran
PowerScore Test Preparation
Follow me on X/Twitter at http://twitter.com/DaveKilloran
My LSAT Articles: http://blog.powerscore.com/lsat/author/dave-killoran
PowerScore Podcast: http://www.powerscore.com/lsat/podcast/
PowerScore Test Preparation
Follow me on X/Twitter at http://twitter.com/DaveKilloran
My LSAT Articles: http://blog.powerscore.com/lsat/author/dave-killoran
PowerScore Podcast: http://www.powerscore.com/lsat/podcast/